Tải bản đầy đủ (.doc) (11 trang)

Chuyên đề bất đảng thức

Bạn đang xem bản rút gọn của tài liệu. Xem và tải ngay bản đầy đủ của tài liệu tại đây (481.43 KB, 11 trang )

BAT ẹANG THệC SCHUR TRONG HèNH HOẽC
Problem 5
The triangle ABC has unequal sides, centroid G, incenter I and orthocenter H. Show that angle GIH > 90o.
Solution
Let N be the midpoint of OH. Then IN = (IO + IH)/2, so IH = 2IN - IO (we use bold to represent vectors). G lies on the
line OH with OG = OH/3 (the Euler line), so OG = 2GN and hence IG = (2IN + IO)/3. Hence IH.IG = (4IN2 - IO2)/3.
We have the well-known results OI2 = R2 - 2rR (Euler's formula and IN = R/2 - r (Feuerbach's theorem - usually stated
as "the incircle and the nine-point circle touch" - N is the center of the nine-point circle and R/2 is the radius of the
nine-point circle).
Hence IH.IG = (R2 - 4Rr + 4r2 - R2 + 2Rr)/3 = -2r(R - 2r)/3 < 0.
If you are fluent with vector formulae for the triangle, the following solution by Mehul Srivastav is straightforward
Use vectors origin O the circumcenter. Take the vector OA to be A etc. Then G = (A + B + C)/3, H = A + B + C (Euler
line), I = (aA + bB + cC)/(a+b+c). The last formula is not so well-known, but is easy to verify. Check, for example, that
b AI.AB = c AI.AC (for that it is convenient to relocate the origin to A).
We have to show that (G-I).(H-I) < 0, or G.H + I2 - I.(G+H) < 0.
Note that since the origin is the circumcenter we have B.C = R2cos2A = R2cos2A - R2sin2A = (R2 - a2/4) - a2/4 = R2 -
a2/2. Similarly for C.A and A.B. Obviously A2 = B2 = C2 = R2. Hence 3G.H = A2 + B2 + C2 + 2A.B + 2B.C + 2C.A =
9R2 - (a2 + b2 + c2).
We have I2 = (aA + bB + cC)2/(a+b+c)2 = ( (a2 + b2 + c2)R2 + 2ab(R2 - c2/2) + 2ac(R2 - b2/2) + 2bc(R2 - a2/2) )/
(a+b+c)2 = R2 - (abc2 + ab2c + a2bc)/(a+b+c)2 = R2 - abc/(a+b+c).
(3/4)(a+b+c)I.(G+H) = (aA + bB + cC).(A + B + C) = (a+b+c)R2 + (a+b)A.B + (b+c)B.C + (c+a)C.A = 3(a+b+c)R2 -
(c2(a+b) + a2(b+c) + b2(a+c))/2.
So we wish to show that 3R2 - (a2+b2+c2)/3 + R2 -abc/(a+b+c) - 4R2 + (2/3)(a2b + b2a + ... )/(a+b+c) < 0, or (a+b+c)
(a2+b2+c2) + 3abc > 2(a2b + b2a + ... ) or a3 + b3 + c3 - (a2b + b2a + ... ) + 3abc > 0 or a(a-b)(a-c) + b(b-a)(b-c) + c(c-
a)(c-b) > 0 (*).
wlog a > b > c. So a(a-c) - b(b-c) > 0. Hence a(a-b)(a-c) + b(b-a)(b-c) > 0. Obviously c(c-a)(c-b) > 0. So (*) holds and he
nce the result.
BT NG THC SCHUR V PHNG PHP I BIN P,Q,R
Vừ Thnh Vn
Lp 11 Toỏn-Khi chuyờn THPT-HKH Hu
*LI M U:


Nh cỏc bn ó bit,bt ng thc Schur l mt bt ng thc mnh v cú nhiu ng dng,tuy nhiờn nú vn cũn khỏ xa l
vi nhiu bn hc sinh THCS cng nh THPT.Qua bi vit ny,tụi mun cng cp thờm cho cỏc bn mt k thut s
dng tt BDT Schur,ú l kt hp vi phng phỏp i bin .
Trc ht tụi xin nhc li v bt ng thc Schur v phng phỏp i bin .
I-BT NG THC SCHUR:
Vi cỏc s thc dng a,b,c v bt kỡ ta luụn cú
Hai trng hp quen thuc c s dng nhiu l k=1 v k=2:
II-PHNG PHP I BIN P,Q,R:
i vi mt s bi bt ng thc thun nht i xng cú cỏc bin khụng õm thỡ ta cú th i bin li nh sau:
t
Ta cú mt s ng thc sau:
.
Đặt
Khi đó
Có thể thấy ngay lợi ích của phương pháp này là mối ràng buộc giữa các biến p,q,r mà các biến a,b,c ban đầu không có như:
Những kết quả trên đây chắc chắn là chưa đủ,các bạn có thể phát triển thêm nhiều đẳng thức,bất đẳng thức liên hệ giữa 3
biến p,q,r.Và điều quan trọng mà tôi muốn nói đến là từ bất đẳng thức và ,ta có:
(từ )
(từ )
Tuy nhiên trong một số trường hợp thì có thể các đại lượng có thể nhận giá trị âm lẫn giá trị dương nên ta thường
sử dụng
.
Có lẽ đến đây các bạn đã hiểu được phần nào về bất đẳng thức Schur và phương pháp đổi biến p,q,r.Sau đây là một số ví dụ
minh họa,nhưng trước hết,các bạn hãy tập làm thử rồi xem đáp án sau:
III-VÍ DỤ MINH HỌA
3.1:Bất đẳng thức Schur :
Ví dụ 1 : Võ Thành Văn :
Cho a,b,c là các số thực không âm.Chứng minh rằng:
Lời giải:
Đặt

Áp dụng BDT Holder,ta có:
Ta cần chứng minh:
(đúng theo BDT Schur)
Vậy ta có đpcm.
Ví dụ 2 : APMO 2004 :
Cho 3 số thực dương .Chứng minh rằng:
Lời giải
Lời giải 1:Khai triển bất đẳng thức trên,ta cần chứng minh:
Ta có:
(theo BDT Schur)
Áp dụng các BDT trên,ta có:
Lời giải 2:
Sử dụng bất đẳng thức AM-GM,ta có:
Bất đẳng thức cuối đã rất quen thuộc,ta có đpcm.
Ví dụ 3 : VMO 2002-Trần Nam Dũng
Chứng minh rằng với mọi ,ta có:
Lời giải:
Sử dụng bất đẳng thức AM-GM,ta có:
Mặt khác sử dụng bất đẳng thức Schur,
Do đó
Bất đẳng thức được chứng minh.
Ví dụ 4 : Arqady
Cho a,b,c là các số không âm,trong đó không có 2 số nào đồng thời bằng 0.Chứng minh rằng:
Lời giải:
Bất đẳng thức cần chứng minh tương đương với
Áp dụng bất đẳng thức Cauchy-Schwarz,ta có:
Áp dụng 2 bất đẳng thức trên,ta có:
Giả sử và đặt .
Ta cần chứng minh
Bất đẳng thức cuối dễ dàng chứng minh bằng cách xét 2 trường hợp: và

Đẳng thức xảy ra khi và .
Ví dụ 5 : Moldova TST 2005 :
Chứng minh rằng nếu a,b,c là các số thực dương và thì:
Lời giải:
Quy đồng mẫu số rồi khai triển,ta cần chứng minh:
Áp dụng bất đẳng thức Schur và giả thiết ,ta có:
Áp dụng BDT AM-GM,ta có:
Mặt khác ta lại có:
Vậy ta có đpcm.
Ví dụ 6 : Vasile Cirtoaje :
Cho là các số thực không âm thỏa mãn .Chứng minh rằng:
Lời giải:
Áp dụng BDT Schur,ta có:

Ta cần chứng minh:
Bất đẳng thức cuối hiển nhiên đúng nên ta có đpcm.
Đẳng thức xảy ra khi .
Ví dụ 7 : Võ Thành Văn :
Cho .Chứng minh rằng:
Lời giải:
Đổi biến theo p,q,r,bât đẳng thức cần chứng minh được viết lại như sau:
( )
Mặt khác,theo BDT Schur,ta có:
Vậy ta có đpcm
Ví dụ 8: Phạm Kim Hùng
Cho a,b,c là các số thực dương thỏa mãn .Chứng minh rằng:
Lời giải:
Quy đồng,rút gọn và đổi biến theo p,q,r,bất đẳng thức cần chứng minh tương đương với
Áp dụng BDT Schur,ta có:
Từ giả thiết

Thay 2 điều trên vào bất đẳng thức cần chứng minh,ta có:
Bất đẳng thức cuối đúng nên ta có đpcm.
Ví dụ 9: CRUX
Cho a,b,c là các số thực không âm thỏa mãn .Chứng minh rằng:
* Bài này đã được anh Hùng sử dụng cho phần BDT TRê-bư-sép trong cuốn Sáng tạo BDT,tuy nhiên bây giờ các bạn sẽ
được thấy một lời giải với BDT Schur và phương pháp đổi biến p,q,r rất tự nhiên.
Lời giải:
Biến đổi bất đẳng thức cần chứng minh và chuyển về dạng ,ta có:
Theo BDT AM-GM thì
Theo BDT Schur,ta có:
Nên ta cần chứng minh:
Vậy BDT được chứng minh.
3.2: Phương pháp đổi biến p,q,r:
Ví dụ 10 : Phạm Kim Hùng
Cho a,b,c là các số thực không âm thỏa mãn a+b+c=3.Chứng minh rằng:
Lời giải:
Quy đồng mẫu số rồi khai triển,ta cần chứng minh:
Sử dụng bất đẳng thức quen biết ,ta có:
Tiếp tục sử dụng bất đẳng thức trên,ta cần chứng minh:
hay
với . Áp dụng BDT AM-GM,ta có nên cần chứng minh:
Bắt đẳng thức cuối hiển nhiên đúng nên ta có đpcm.
Đẳng thức xảy ra khi hoặc và các hoán vị.
Ví dụ 11 : Dương Đức Lâm
Cho a,b,c > 0.Chứng minh rằng
Lời giải: Võ Quốc Bá Cẩn
Đặt bất đẳng thức cần chứng minh tương đương với
Áp dụng BDT Cauchy-Schwarz,ta có

×